Đến nội dung

Hình ảnh

Topic ôn luyện VMO 2015

vmo2015

  • Please log in to reply
Chủ đề này có 137 trả lời

#61
Karl Heinrich Marx

Karl Heinrich Marx

    Sĩ quan

  • Thành viên
  • 321 Bài viết

Bài ở trên mình đưa ra ý tưởng vậy nhưng mà mình nói không chắc về kq và đúng là mình nhầm rồi, kết quả chu kì là 7 như em ở dưới làm mới chính xác nhé.

Ngoài ra đề bài 31 và 32 có nhầm chỗ nào không? Đề bài 31, $l$ đóng vai trò gì? Đề bài 32 nếu $Q$ bậc $n+1$ và có đúng $n$ nghiệm phân biệt thì $Q$ có 1 nghiệm kép rồi khi đó tồn tại $b_i$ mà $Q'(b_i)=0$, mâu thuẫn với điều kiện $P(b_i)Q'(b_i)<0$.



#62
LNH

LNH

    Bất Thế Tà Vương

  • Hiệp sỹ
  • 581 Bài viết

 

Bài 2: Tô tập số nguyên bởi $4$ màu. $x,y$ là số nguyên lẻ thỏa mãn $\left | x \right |$ khác $\left | y \right |$ . CMR: tồn tại 2 số nguyên cùng màu có hiệu thuộc ${x,y,x+y,x-y}$.

 

Giả sử phản chứng: tồn tại một cách tô tập $Z$, $f: \mathbb{Z}\rightarrow \left \{ X,D,T,V \right \}$ sao cho với mọi $a \in Z$, ta có:

$\left \{ f\left ( a \right ),f\left ( a+x \right ),f\left ( a+y \right ),f\left ( a+x+y \right ) \right \}=\left \{ X,D,T,V \right \}$

Xét $g:\mathbb{Z}^2\rightarrow \left \{ X,D,T,V \right \}$ sao cho $g\left ( i,j \right )=f\left ( ix+jy \right )$

Biểu diễn các điểm này trên mạng lưới nguyên

4 đỉnh của một hình vuông đơn vị đôi một khác màu

Xét một cột bất kì

Nếu tồn tại $3$ đỉnh liên tiếp trên cột khác màu thì cách tô trên t/m:

+)Mỗi hàng chỉ có $2$ màu

+)Nếu $2$ hàng cách nhau một số chẵn thứ tự thì cùng tập màu (*)

Nếu một cột bất kì không tồn tại $3$ ô liên tiếp khác màu khì (*) sẽ đúng cho các cột. Không mất tính tổng quát, giả sử (*) đúng.

Giả sử hàng $y=0$ được tô màu D,V

Nếu $g\left ( 0,0 \right )=D\Rightarrow g\left ( 0,y \right )=V,g\left ( x,0 \right )=V$

$\Rightarrow g\left ( 0,x \right )=f\left ( xy \right )=g\left ( y,0 \right )=V$

Mặt khác,  hàng chứa $\left ( 0,x \right )$ là hàng thứ $x$, hàng chứ $\left ( y,0 \right )$ là hàng thứ 0, suy ra vô lí

Vậy ta có đpcm



#63
WhjteShadow

WhjteShadow

    Thượng úy

  • Phó Quản lý Toán Ứng dụ
  • 1323 Bài viết

 

Bài 31: Cho $a_1,a_2,a_3...$ là một dãy số thực dương. Giả sử với một số nguyên dương $s$ cho trước, ta có $$a_n=\max\{a_k.a_{n-k}:1\le k\le n-1\}$$

với mọi $n>s$. Chứng minh rằng tồn tại các số nguyên dương $l$ và $N$, với $l\le s$ thỏa mãn $a_n=a_l.a_{n-l} \forall n\ge N$.

 

Đặt $b_n=\ln a_n$, ta có bài toán IMO2010 P6 :

Cho $b_1,b_2,b_3,..,b_{s},...$ là dãy số thực dương. Giả sử với 1 số nguyên dương $s$ cho trước, ta có $b_n=\max\{b_k+b_{n-k}:1\leq k \leq n-1\}\forall n>s$. Chứng minh rằng tồn tại $l,N$, $l\leq s$ thỏa mãn $b_n=b_{l}+b_{n-l}\forall n\ge N$.

Giải như sau :

Với mọi $n>s$, ta có thể phân tích $b_n=b_{i_{1}}+b_{i_{2}}$ với $i_1+i_2=n, 1\leq i_1,i_2\leq n-1$, nếu $i_1$ hoặc $i_2>s$, ta lại tiếp tục phân tích như vậy, tóm lại ta có thể viết $b_n$ dưới dạng :

$$b_n=b_{i_1}+b_{i_2}+...+b_{i_{k}}\,\,\,\,(1\leq i_j\leq s\forall j=\overline{1;k}, \, i_1+i_2+...+i_k=n)$$

Gọi $l\in \{1;2;...;s\}$ là 1 chỉ số sao cho $\frac{a_l}{l}=r=\max_{1\leq i\leq s} \frac{a_i}{i}$.

Xác định dãy $\{x_n\}$ : $x_n=b_n-rn\forall n\in \mathbb{Z}^{+}$, lúc đó $x_l=0$ và ta có :

$$x_n=b_n+rn=\max_{1\leq k\leq n-1}(b_k+b_{n-k})+rn=\max_{1\leq k\leq n-1}(b_k+b_{n-k}+kr+(n-k)r)$$

$$=\max_{1\leq k\leq n-1} (x_k+x_{n-k})$$

Từ đó dễ dàng suy ra $x_n\leq 0\forall n\in\mathbb{Z}^{+}$ và $$x_n=x_{i_1}+x_{i_2}+...+x_{i_{k}}\,\,\,\,(1\leq i_j\leq s\forall j=\overline{1;k}, \, i_1+i_2+...+i_k=n)$$

$\bullet$ Nếu $x_k=0\forall k\in {1;2;..;s}$ thì $x_n=0\forall n\in\mathbb{Z}^{+}$ hiển nhiên có đpcm.

$\bullet$ Nếu tồn tại $k \, : \, x_k\neq 0$ với $k\in {1;2;...;s}$. ta đặt 

$$M=\max_{1\leq k\leq s} |x_k|, m=\min_{1\leq k\leq s} |x_k| \,\,(x_k<0)$$

Lúc đó $\forall n>s$, ta có : $x_n=\max{x_k+x_{n-k}}\ge x_{n-l}+x_l=x_{n-l}$, vậy nên :

$$0\geq x_n\geq x_{n-l}\geq ...\geq x_{n-l.\left[\frac{n}{l}\right]}\geq -M$$

(Do $l\leq s$ nên $n-l.\left[\frac{n}{l}\right]\leq s$, lúc đó $x_{n-l.\left[\frac{n}{l}\right]}\geq -M$)

Vậy tóm lại $x_n\in [-M;0]\forall n\in \mathbb{Z}^{+}$. Mặt khác ta sẽ chứng minh tập sau chỉ có hữu hạn giá trị :

$$\{x_{i_1}+x_{i_2}+...+x_{i_{k}}\} \cap [0;-M]\,\,\,\forall 1\leq i_j\leq s$$

Thật vậy gọi $t$ là số chỉ số $i$ sao cho $x_i \neq 0$, lúc đó $x_{i_1}+x_{i_2}+...+x_{i_{k}}\leq t.m\Rightarrow -M\leq t.m \Rightarrow t\leq \frac{M}{m}$.

Vậy chỉ có hữu hạn chỉ số $i$ sao cho $x_i \neq 0$ mà chỉ có $s$ giá trị của các $x_i\,\, \Rightarrow $ có hữu hạn giá trị $\{x_{i_1}+x_{i_2}+...+x_{i_{k}}\} \cap [0;-M]\,\,\,\forall 1\leq i_j\leq s$ hay chỉ có hữu hạn giá trị $x_n\, n\in \mathbb{Z}^{+}$.

Xét dãy $x_n,x_{n+l},x_{n+2l},.....$ là 1 dãy giảm bị chặn dưới và có hữu hạn giá trị $\Rightarrow $ nó là dãy hằng kể từ 1 chỉ số đủ lớn nào đó. Cho $n$ chạy qua hệ thặng dư đầy đủ mod $l$, thì dễ thấy $(x_n)$ là dãy tuần hoàn chu kì $l$ kể từ 1 giá trị $N$ đủ lớn. Lúc đó :

$$x_n=x_{n-l}=x_{n-l}+x_l\forall n>N+l$$

$$\Rightarrow b_n=b_{n-l}+b_l\forall n>N+l$$

Kết thúc chứng minh.

 

=========================================================================

Vậy là đã giải quyết gần xong đống bài còn tồn đọng, từ giờ anh Nam bận 1 thời gian nên topic sẽ do các ĐHV Olympic và tổng hợp quản lí. Các bạn có thể post bài mới và tham gia thảo luận tiếp tục từ đây.

Chú ý là chúng ta nên post bài đúng với trọng tâm của đề thi quốc gia những năm gần đây như pt, hpt, dãy số, hình học, tổ hợp (đặc biệt là tổ hợp số học hoặc số học tổ hợp hoặc hình học tổ hợp).

Vậy thôi, chúc các bạn ôn thi VMO tốt hơn với topic.

P/s : Đã sửa đề bài 32.


Bài viết đã được chỉnh sửa nội dung bởi WhjteShadow: 19-11-2014 - 17:47

“There is no way home, home is the way.” - Thich Nhat Hanh

#64
Ispectorgadget

Ispectorgadget

    Nothing

  • Quản lý Toán Phổ thông
  • 2946 Bài viết

Bài 34: Cho dãy hữu hạn $a_0,a_1,\cdots,a_N$ thỏa mãn $a_1=a_N=0$ và $$a_{i+1}-2a_i+a_{i-1}=a_i^2,\, \forall i=1,2,3,\cdots,N-1$$

Chứng minh $a_i\le 0,\; \forall i=0,1,2,\cdots,n$.

►|| The aim of life is self-development. To realize one's nature perfectly - that is what each of us is here for. ™ ♫


#65
LNH

LNH

    Bất Thế Tà Vương

  • Hiệp sỹ
  • 581 Bài viết

Bài 35: Cho $2$ tập hợp $T$ và $P$, $T$ chứa $66$ điểm, $P$ chứa $16$ đường thẳng. $(A,l)$ được gọi là tốt nếu $A \in T$, $l \in P$, $A \in l$. Tìm số bộ tốt lớn nhất.



#66
Karl Heinrich Marx

Karl Heinrich Marx

    Sĩ quan

  • Thành viên
  • 321 Bài viết

Bài 32: Nếu $Q(x)$ bậc $n+1$ mà có $n+1$ nghiệm phân biệt $b_1,b_2,..,b_{n+1}$ thì ta có $Q'(b_i)Q'(b_{i+1})<0$ với mọi $i=1,2,..,n$ bởi vì một đồ thị liên tục mà cắt trục hoành 2 lần liên tiếp thì sẽ phải có một lần đi xuống một lần đi lên. Từ đó suy ra $P(b_i)P(b_{i+1}<0$ với $i=1,2,..,n$

Từ đây có thể thấy luôn là $P(x)$ phải có bậc $n$ và có $n$ nghiệm phân biệt.

Bài 33: Một dãy số có dạng $u_{n+1}-ku_n+u_{n-1}=0$ thì có tính chất $u_{n+1}u_{n-1}-u_n^2=c$ với $c$ là hằng số. Dãy này mà tuần hoàn thì $u_{n+1}u_{n-1}-u_{n}^2$ cũng tuần hoàn do vậy tìm k sao cho c=1 hoặc c=-1 hoặc $u_2u_0-u_1^2=0$ sau đấy kiểm tra lại kq.

Bài 34: Có thể xem như tính chất lồi lõm của dãy số. Đặt $g(i)=a_{i}-a_{i-1}$ (cái này xem như đạo hàm bậc nhất). Đầu đề bài tương đương với $g(i)$ tăng mà $g(2)+g(3)+..+g(N)=a_N-a_1=0$ có thể suy ra ngay $g(2)+g(3)+...+g(i) \le 0$ với mọi $i \le N$ tức là $a_i-a_1 \le 0$ với mọi $i$ đây chính là điều cần chứng minh


Bài viết đã được chỉnh sửa nội dung bởi namcpnh: 22-11-2014 - 07:48
Latex


#67
WhjteShadow

WhjteShadow

    Thượng úy

  • Phó Quản lý Toán Ứng dụ
  • 1323 Bài viết

Bài 35 : Xét 1 đồ thị với 16 đỉnh $A_1,A_2,...,A_{16}$ và 66 đỉnh $B_1,B_2,...,B_{66}$. $A_i$ được nối với $B_j$ khi và chỉ khi điểm $j$ nằm trên đường thẳng $i$ (hay $(j;i)$ là "tốt"). 

v1.png

Gọi $d_i$ là số các cạnh có 1 đầu mút là $B_i$ với $i=\overline{1;66}$. Dễ thấy số bộ tốt chính bằng số đoạn thẳng trên đồ thị và bằng $\sum^{66}_{i=1} d_i$. 

Ta sẽ đếm số góc dạng $\widehat{A_iB_kA_j}$ theo 2 cách.

Đầu tiên với 2 điểm $A_i,A_j$ bất kì, chỉ tồn tại nhiều nhất 1 điểm $B_k$ được nối với $A_i,A_j$ (và tạo thành 1 góc), thật vậy nếu giả sử có thêm góc $\widehat{A_iB_mA_j}$, lúc đó đường thẳng $k$ và $m$ cùng chứa 2 điểm $i;j$ (vô lí vì 2 đường thẳng này không trùng nhau).

v2.png

Vậy số góc nhiều nhất được tạo thành là $C^2_{16}$.

Mặt khác với mỗi điểm $B_i$, có $d_i$ cạnh nhận $B_i$ là đầu mút, vậy có $C^2_{d_i}$ góc nhận $B_i$ là đỉnh.

Từ 2 cách trên, ta có $120=C^2_{16}\geq \sum^{66}_{i=1} C^2_{d_i}=\frac{\sum^{66}_{i=1}d_i^2-\sum^{66}_{i=1}d_i}{2}\\ \geq \dfrac{\frac{(\sum^{66}_{i=1} d_i)^2}{66}-\sum^{66}_{i=1}d_i}{2}$

Từ đó, giải bất phương trình ta có $\sum^{66}_{i=1}d_i\leq 163$. Xét 1 vài TH :

$\bullet$ Nếu $\sum^{66}_{i=1}d_i=163$

Lúc đó gọi $d_m=\min\{d_i|i=\overline{1;66}\}$, $d_n=\max\{d_i|i=\overline{1;66}\}$ thì dễ dàng có $2\leq d_m\leq d_n\leq 3$. Gọi $a$ là số số $d_i$ có giá trị bằng 2, thì $66-a$ là số số $d_i$ có giá trị bằng 3. Lúc đó $2a+3(66-a)=163\Rightarrow a=35$ suy ra 

$\sum^{66}_{i=1}d_i^2=4a+9(66-a)=419$ nhưng lúc đó $\frac{\sum^{66}_{i=1}d_i^2-\sum^{66}_{i=1}d_i}{2}>120$ (mâu thuẫn).

Tương tự TH $162,161,160$ ta cũng có điều vô lí.

Ta sẽ chỉ ra với $\sum^{66}_{i=1}d_i=159$ thì có cấu hình thỏa mãn đề bài, lúc đó bằng tính toán tương tự trên, ta có 39 điểm $d_i=2$, 27 điểm có $d_j=3$. Lúc đó với $16$ đường thẳng của mình, ta sẽ vẽ sao cho có $27$ bộ 3 đường thẳng đồng quy và tất cả các đường thẳng đôi một cắt nhau, vậy ngoài 27 điểm thuộc 3 đường thẳng như trên, số giao điểm của chỉ đúng 2 đường thẳng là $C^2_{16}-27.3=39$. 

Vậy $159$ là số bộ tốt nhất cần tìm !

 

 

Bài 33: Một dãy số có dạng $u_{n+1}-ku_n+u_{n-1}=0$ thì có tính chất $u_{n+1}u_{n-1}-u_n^2=c$ với $c$ là hằng số. Dãy này mà tuần hoàn thì $u_{n+1}u_{n-1}-u_{n}^2$ cũng tuần hoàn do vậy tìm k sao cho c=1 hoặc c=-1 hoặc $u_2u_0-u_1^2=0$ sau đấy kiểm tra lại kq.

 

@@~ Em vẫn không hiểu đoạn dãy $u_{n+1}.u_{n-1}-u_n^2$ tuần hoàn thì tại sao $c=1$ hoặc $-1$ ạ.

Bài này em làm như thế này : Với $k>2$ hoặc $k<-2$ ta đều chứng minh đc trị tuyệt đối của các số hạng đơn điệu tăng (hoặc giảm) nên không tuần hoàn. 

Với $|k|\leq 2$ thì phương trình đặc trưng có nghiệm phức $\frac{k+i.\sqrt{4-k^2}}{2}$, đặt $\frac{k}{2}=\cos a$ với $a\in [0;\pi]$ thì ta có số hạng tổng quát của dãy là $u_n=A.\cos na+B.\sin na$ với $A,B$ xác định được từ $u_0,u_1$.

Và lúc đó giả sử dãy tuần hoàn chu kì $l$, thì $A.\cos na+B.\sin na=A.\cos (n+l)a+B.\sin (n+l)a\forall n\in\mathbb{N}$

$\Leftrightarrow \frac{A}{\sqrt{A^2+B^2}}.\cos na+\frac{B}{\sqrt{A^2+B^2}}.\sin na=\frac{A}{\sqrt{A^2+B^2}}.\cos (n+l)a+\frac{B}{\sqrt{A^2+B^2}}.\sin (n+l)a\forall n\in\mathbb{N}$

$\Leftrightarrow \sin(na+\alpha)=\sin((n+l)a+\alpha)$ với $\frac{A}{\sqrt{A^2+B^2}}=\cos \alpha,\frac{B}{\sqrt{A^2+B^2}}=\sin \alpha$

$\Leftrightarrow la= 2h\pi$ với $h\in\mathbb{Z}\Leftrightarrow a=\frac{2h\pi}{l}$. Vậy $k=\cos \left(\frac{2h\pi}{l}\right)$, đến đây tìm $k$ hữu tỉ ... 

----------------------

Bài 36 :

Cho $a,b,c$ là các hằng số, $\sqrt{a}+\sqrt{b}+\sqrt{c}=\frac{\sqrt{3}}{2}$. Chứng minh rằng hệ sau có nghiệm duy nhất :

$\left\{\begin{matrix} \sqrt{x-b}+\sqrt{x-c}=1\\ \sqrt{y-c}+\sqrt{y-a}=1\\ \sqrt{z-a}+\sqrt{z-c}=1 \end{matrix}\right.$


Bài viết đã được chỉnh sửa nội dung bởi WhjteShadow: 22-11-2014 - 18:10

“There is no way home, home is the way.” - Thich Nhat Hanh

#68
Karl Heinrich Marx

Karl Heinrich Marx

    Sĩ quan

  • Thành viên
  • 321 Bài viết

 

@@~ Em vẫn không hiểu đoạn dãy $u_{n+1}.u_{n-1}-u_n^2$ tuần hoàn thì tại sao $c=1$ hoặc $-1$ ạ.

Bài này em làm như thế này : Với $k>2$ hoặc $k<-2$ ta đều chứng minh đc trị tuyệt đối của các số hạng đơn điệu tăng (hoặc giảm) nên không tuần hoàn. 

Với $|k|\leq 2$ thì phương trình đặc trưng có nghiệm phức $\frac{k+i.\sqrt{4-k^2}}{2}$, đặt $\frac{k}{2}=\cos a$ với $a\in [0;\pi]$ thì ta có số hạng tổng quát của dãy là $u_n=A.\cos na+B.\sin na$ với $A,B$ xác định được từ $u_0,u_1$.

Và lúc đó giả sử dãy tuần hoàn chu kì $l$, thì $A.\cos na+B.\sin na=A.\cos (n+l)a+B.\sin (n+l)a\forall n\in\mathbb{N}$

$\Leftrightarrow \frac{A}{\sqrt{A^2+B^2}}.\cos na+\frac{B}{\sqrt{A^2+B^2}}.\sin na=\frac{A}{\sqrt{A^2+B^2}}.\cos (n+l)a+\frac{B}{\sqrt{A^2+B^2}}.\sin (n+l)a\forall n\in\mathbb{N}$

$\Leftrightarrow \sin(na+\alpha)=\sin((n+l)a+\alpha)$ với $\frac{A}{\sqrt{A^2+B^2}}=\cos \alpha,\frac{B}{\sqrt{A^2+B^2}}=\sin \alpha$

$\Leftrightarrow la= 2h\pi$ với $h\in\mathbb{Z}\Leftrightarrow a=\frac{2h\pi}{l}$. Vậy $k=\cos \left(\frac{2h\pi}{l}\right)$, đến đây tìm $k$ hữu tỉ ... 

 

Lúc đấy viết như thế nhưng nghĩ trong đầu lại là $u_{n+1}u_{n-1}-u_n^2=c(u_nu_{n-2}-u_{n-1}^2)$ em ạ. Dạo này làm cái gì cũng sai cả.

Giải như em là chính xác rồi nhưng hình như đoạn tìm $k$ hữu tỉ ko biết có đơn giản ko nhể. Đoạn 2 cái sin bằng nhau cần lập luận 1 chút thì phải vì $sin(x)$ vẫn có thể bằng $sin( \pi -x)$ nữa

Có thể diễn đạt gọn hơn 1 chút thế này.

Ta có thể viết $u_{n+1}-au_n=b(u_n-au_{n-1})$ với $a,b$ thỏa mãn $ab=1,a+b=k$ tức thỏa mãn phương trình $k^2-k+1=0$ (Cái này giải thích cho lí do có phương trình đặc trưng)

Dãy $u_n$ tuần hoàn thì khi đó hoặc $u_1-au_0=0 \rightarrow a=-1$ hoặc tồn tại $l$ mà $b^l=1$

Nếu $k>2$ thì 2 nghiệm $a,b$ không thỏa 2 điều trên được.

Nếu $k=2$ hoặc $k=-2$ thì đều ok.

Nếu $|k|<2$ thì khi đó ta chọn nghiệm $b= \frac{k+i\sqrt{4-k}}{2}$ đặt $\frac{k}{2}= cos(t)$

Khi đó thì $b=cos(t)+isin(t) \Rightarrow b^l=r(cos(t)+isin(t))^l=cos(lt)+isin(lt)=1 \Rightarrow lt=2h\pi$ (không hiểu vì sao sửa mãi dòng này mà chẳng hiện công thức, bạn mod nào sửa dùm mình nhá, một vài lần gần đây mình đánh công thức kiểm tra có vẻ không sai nhưng chẳng hiện là sao nhỉ)

Vào lúc 21 Tháng 11 2014 - 23:31, WhjteShadow đã nói:

 

$\bullet$ Nếu $\sum^{66}_{i=1}d_i=163$

Lúc đó gọi $d_m=\min\{d_i|i=\overline{1;66}\}$, $d_n=\max\{d_i|i=\overline{1;66}\}$ thì dễ dàng có $2\leq d_m\leq d_n\leq 3$. Gọi $a$ là số số $d_i$ có giá trị bằng 2, thì $66-a$ là số số $d_i$ có giá trị bằng 3. Lúc đó $2a+3(66-a)=163\Rightarrow a=35$ suy ra 

$\sum^{66}_{i=1}d_i^2=4a+9(66-a)=419$ nhưng lúc đó $\frac{\sum^{66}_{i=1}d_i^2-\sum^{66}_{i=1}d_i}{2}>163$ (mâu thuẫn).

Tương tự TH $162,161,160$ ta cũng có điều vô lí.

 

Hình như $\frac{419-163}{2}<163$ mà em.

Em trai Whjte Shadow phong độ giải toán bữa nay ổn định quá, cố gắng 1 suất IMO em nhá, đầu tư hình vào em ạ, thành bại đều vì hình.

----------

@WS : Hì hì đoạn đó là 120 ạ, em gõ vội quá nên nhầm. Còn đoạn trên em gõ tắt quá, nên thành ra thiếu. Mà đến đó tìm $k$ để cos hữu tỉ chắc là khó ạ @@


Bài viết đã được chỉnh sửa nội dung bởi WhjteShadow: 22-11-2014 - 18:12


#69
LNH

LNH

    Bất Thế Tà Vương

  • Hiệp sỹ
  • 581 Bài viết

Bài 37: Cho các số nguyên $x,y,z$ với $x>2,y>1,z>0$ thỏa mãn:

$x^y+1=z^2$

Gọi $p$ là số ước nguyên tố phân biệt của $x$, $q$ là số ước nguyên tố phân biệt của $y$. Chứng minh rằng: $p \geq q+2$


Bài viết đã được chỉnh sửa nội dung bởi namcpnh: 09-12-2014 - 23:50


#70
pndpnd

pndpnd

    Trung sĩ

  • Thành viên
  • 164 Bài viết

Bài 38: Tìm hàm liên tục $f: R\rightarrow R$  thoả mãn:

  1) $f$ là đơn ánh

  2) $f(2x-f(x))=x$

  3) Tồn tại $x_{0}$ thỏa mãn $f(x_{0})=x_{0}$



#71
BlackSelena

BlackSelena

    $\mathbb{Sayonara}$

  • Hiệp sỹ
  • 1549 Bài viết
Bài 36 :

Cho $a,b,c$ là các hằng số, $\sqrt{a}+\sqrt{b}+\sqrt{c}=\frac{\sqrt{3}}{2}$. Chứng minh rằng hệ sau có nghiệm duy nhất :

$\left\{\begin{matrix} \sqrt{x-b}+\sqrt{x-c}=1\\ \sqrt{y-c}+\sqrt{y-a}=1\\ \sqrt{z-a}+\sqrt{z-c}=1 \end{matrix}\right.$

http://diendantoanho...21/#entry387173

Chú ý thêm là giao điểm của 2 đường thẳng đó chỉ cắt trong tam giác $ABC$ tại 1 điểm, nên đó là nghiệm duy nhất phương trình

 

Bài 26:

Cho tam giác $ABC$, $M$ là 1 điểm trong tam giác. $AM, BM, CM$ cắt $BC, CA, AB$ tại $D, E, F$. $(AEF)$ cắt $(O)$ tại $A_1$. Tuơng tự có $B_1, C_1$. $X \equiv B_1C_1 \cap BC$, tương tự có $Y, Z$.

Chứng minh $O$ là trực tâm $\triangle XYZ$

Hình vẽ...

Bổ đề 1:

Bổ đề 2:

Lời giải:


Bài viết đã được chỉnh sửa nội dung bởi BlackSelena: 22-11-2014 - 15:57


#72
maths281997

maths281997

    Binh nhì

  • Thành viên
  • 10 Bài viết

Bài 39: Tìm tất cả các số nguyên dương n sao cho tồn tại các số thực phân biệt khác 0 : $a_{1},a_{2},...a_{n}$ mà:

$\left \{ a_{i}+\frac{\left ( -1 \right )^{i}}{a_{i}} |1\leq i\leqslant n\right \}=\left \{ a_{i}|1\leq i\leq n \right \}$

Đề Turkey National Olympiad Second Round 2014 bài 5

 

Bài viết đã được chỉnh sửa nội dung bởi LNH: 22-11-2014 - 19:48


#73
WhjteShadow

WhjteShadow

    Thượng úy

  • Phó Quản lý Toán Ứng dụ
  • 1323 Bài viết

Bài 38: Tìm hàm liên tục $f: R\rightarrow R$  thoả mãn:

  1) $f$ là đơn ánh

  2) $f(2x-f(x))=x$

  3) Tồn tại $x_{0}$ thỏa mãn $f(x_{0})=x_{0}$

Mình làm hơi loằng ngoằng chút mà ý tưởng ban đầu là thế này :

Do $f$ liên tục và đơn ánh nên dễ thấy $f$ đơn điệu.

Đặt $g(x)=2x-f(x)\forall x\in \mathbb{R}$, lúc đó $g$ cũng liên tục và $g(x_0)=x_0$. Mặt khác ta có với $g(x)=g(y)$ thì :

$$f(g(x))=f(g(y))\,\,\,\Rightarrow\,\,\, x=y$$

Vậy $g$ cũng đơn ánh, ta cũng có $g$ đơn điệu, từ $2)$ suy ra :

$$f(g(x))=x\,\,\,\Rightarrow \,\,\, 2g(x)-g(g(x))=x \\ \Rightarrow g(g(x))-g(x)=g(x)-x,\,\,\,\, (*)$$

Kí hiệu $g^{(n)}(x)=g(g(....(x)...))$ ( $n$ dấu $($ và $)$ ), lúc đó dễ thấy $g^{(n)}(x)-g^{(n-1)}(x)=g^{(n-1)}(x)-g^{(n-2)}(x)=.....=g(x)-x$, nên $$g^{(n)}(x)-x=n.(g(x)-x)\,\,\,\Rightarrow \,\,\, g^{(n)}(x)=n.(g(x)-x)+x$$

$\bullet$ TH1 : Nếu $g$ đơn điệu giảm, lúc đó với $x<y$ thì $g^{(2k+1)}(x)>g^{(2k+1)}(y),\, g^{(2k)}(x)<g^{(2k)}(y) \,\,\forall k\in \mathbb{Z}^{+}$

$\Rightarrow \,\, (2k+1).(g(x)-x)+x > (2k+1).(g(y)-y)+y \,\, , \,\,  2k.(g(x)-x)+x < 2k.(g(y)-y)+y\,\,\forall k\in \mathbb{Z}^{+}$

Thành thử ra $g(x)-x=g(y)-y$ (Vì nếu $g(x)-x<g(y)-y$ thì bđt thứ nhất sai với $k$ đủ lớn, ngược lại thì bđt 2 sai với $k$ đủ lớn).

Vậy $g(x)-x=g(y)-y\forall x,y\in \mathbb{R}$ mà tồn tại $x_0$ để $g(x_0)=x_0$ nên $g(x)=x\forall x\in \mathbb{R}$ từ đó ta có $f$.

$\bullet$ TH2 : Nếu đơn điệu tăng, làm tương tự như trên thì với $x<y$ ta luôn có $k.(g(x)-x)+x <k.(g(y)-y)+y\,\,\forall k\in \mathbb{Z}^{+}$, từ đó suy ra $g(x)-x\leq g(y)-y\forall x<y$.

Đặt $h(x)= g(x)-x\forall x\in \mathbb{R}$ thì $h(x)\leq h(y)\forall x<y, \,h(x_0)=0$ và $h$ liên tục. Dễ thấy $h(x)\geq 0\forall x>x_0$, $h(x)\leq 0\forall x<x_0$.Từ $(*)$ ta có :

$$h(x)=h(x+h(x))=h(x+h(x)+h(x+h(x))=h(x+2h(x))=\\ =.....=h(x+k.h(x))\forall k\in\mathbb{Z}^{+}, x\in\mathbb{R}$$

Giả sử tồn tại $x_1>x_0$ để $h(x_1)>0$. Vậy thì $h(x_1)=h(x_1+h(x_1))=h(x_1+2h(x_1))=.....=h(x_1+k.h(x_1))\forall k\in\mathbb{Z}^{+}$, lúc đó với $x>x_1$ bất kì, tồn tại $T$ sao cho $x_1+T.h(x_1)>x$ và do $h$ là hàm không giảm nên $h(x_1)\leq h(x)\leq h(x_1+T.h(x_1))\Rightarrow h(x_1)=h(x)= h(x_1+T.h(x_1))$

Vậy $h(x)=h(x_1)\forall x>x_1$. Mặt khác do $h$ liên tục nên tồn tại $x_2\in (x_0;x_1)$ và $0<h(x_2)<h(x_1)$. Làm tương tự trên, ta lại có $h(x)=h(x_2)\forall x>x_1$, từ 2 điều trên suy ra mâu thuẫn.

Vậy $h(x)=0\forall x\geq x_0$. Tương tự $h(x)=0\forall x\leq x_0$

Vậy tóm lại $h(x)=0\forall x\in \mathbb{R}$ hay $f(x)=x\forall x\in \mathbb{R}$.

 

=====================================

 

Bài 40 : Tìm hàm $f$ liên tục $\mathbb{R}\to \mathbb{R}$ sao cho $f(x)-f(y)\in \mathbb{Q}\forall x-y \in \mathbb{Q}$

Bài 41 : Cho tam giác $ABC$ nội tiếp đường tròn $(O)$. $H$ là hình chiếu của $A$ xuống $BC$, $AK$ là đường kính của $(O)$. $I$ và $J$ lần lượt là tâm đường tròn nội tiếp và bàng tiếp góc $A$ của $\Delta \,ABC$. Chứng minh rằng $\widehat{BIH}=\widehat{CIK}$ và $\widehat{BJH}=\widehat{CJK}$


Bài viết đã được chỉnh sửa nội dung bởi WhjteShadow: 23-11-2014 - 12:38

“There is no way home, home is the way.” - Thich Nhat Hanh

#74
banhgaongonngon

banhgaongonngon

    Thượng úy

  • Thành viên
  • 1046 Bài viết

Bài 38: Tìm hàm liên tục $f: R\rightarrow R$  thoả mãn:

  1) $f$ là đơn ánh

  2) $f(2x-f(x))=x$

  3) Tồn tại $x_{0}$ thỏa mãn $f(x_{0})=x_{0}$

 

Xét hàm $g: \mathbb{R} \rightarrow \mathbb{R}$, $g(x)=2x-f(x)$
 

Từ $(2)$ ta có $g(g(x))=2g(x)-f(g(x))=2g(x) -x$

 

Do đó $g$ là một đơn ánh và vì $g$ liên tục nên $g$ đơn điệu thực sự

 

Trường hợp 1:  $g$ giảm thực sự. Ta có

$g(g(x)) - g(x) = g(x) - x$  $(4)$

 

Nếu $g(x) > x$ thì $g(g(x) < g(x)$, mâu thuẫn với $(4)$, tương tự ta cũng gặp mâu thuẫn nếu $g(x)<x$

 

Do đó $g(x)=x,\forall x\in \mathbb{R}$, không thể có vì $g$ giảm thực sự

 

Trường hợp 2: $g$ tăng thực sự.

 

Từ $g(g(x))=2g(x)-x$, bằng quy nạp ta có thể chứng minh $g_{n}\left ( x \right )=ng\left ( x \right )-(n-1)x,\forall x\in \mathbb{R}$

 

Đặc biệt $ g_{n}\left ( 0 \right )=ng\left ( 0 \right )$

 

Do đó

$g_{n}(x)-g_{n}(0)=ng(x)-ng(0)-(n-1)x,\forall x\in \mathbb{R}$

hay

$g(x)-g(0)-x=\frac{g_{n}(x)-g_{n}(0)-x}{n},\forall x\in \mathbb{R}$   $(5)$

 

Bởi $g$ tăng thực sự nên khi cho $n\rightarrow \propto $ ta được

$\left\{\begin{matrix} g\left ( x \right )\geq x+g(0),\forall x \geq 0\\ g(x)\leq x+g(0),\forall x < 0 \end{matrix}\right.$

 

Ta sẽ chứng minh $g$ là một toàn ánh

 

Thật vậy đặt $m= \inf\left \{ g(x)\mid x \in \mathbb{R} \right \}$ và $M= \sup \left \{ g(x)\mid x \in \mathbb{R} \right \}$

 

Giả sử $M < \propto$, khi đó chọn $x=M+a - g(0) > 0$ với $a > 0$ tùy ý ta được

 

$g(M+a-g(0))\geq M+a>M$, mâu thuẫn

 

Chứng tỏ $M = \propto$, tương tự $m= - \propto$, nên $g$ là một toàn ánh

 

$g$ đơn ánh và toàn ánh nên là song ánh, như vậy tồn tại ánh xạ ngược $g^{-1}$

 

Ta thấy $g^{-1}$ cũng tăng thực sự

 

Từ $(5)$ cho $n\rightarrow -\propto $ ta có 

$\left\{\begin{matrix} g\left ( x \right )\leq x+g(0),\forall x \geq 0\\ g(x)\geq x+g(0),\forall x < 0 \end{matrix}\right.$

 

Vậy $g(x) = x+g(0), \forall x \in \mathbb{R}$  và $f(x) = x-g(0)=x+f(0), \forall x\in \mathbb{R}$

 

Vì tồn tại $x_{0}$ sao cho $f(x_{0})=x_{0}$ nên $f(x) = x, \forall x \in \mathbb{R}$


Bài viết đã được chỉnh sửa nội dung bởi namcpnh: 23-11-2014 - 16:48


#75
LNH

LNH

    Bất Thế Tà Vương

  • Hiệp sỹ
  • 581 Bài viết

Bài 41 : Cho tam giác $ABC$ nội tiếp đường tròn $(O)$. $H$ là hình chiếu của $A$ xuống $BC$, $AK$ là đường kính của $(O)$. $I$ và $J$ lần lượt là tâm đường tròn nội tiếp và bàng tiếp góc $A$ của $\Delta \,ABC$. Chứng minh rằng $\widehat{BIH}=\widehat{CIK}$ và $\widehat{BJH}=\widehat{CJK}$

Capture.PNG

Giả sử $AC > AB$

Gọi $D$ là giao điểm $AI$ và đường tròn tâm $O$

Ta cần chứng minh $\angle BIH=\angle CIK\Leftrightarrow \angle DIB-\angle DIC=\angle HID-\angle KID$

$\angle IBC-\angle ICB=\angle HID-\angle KID$

Gọi $T$ sao cho $TI \perp BI$ với $T \in AH$

Ta có: $\angle IBC=\angle HTI=\angle HAI+\angle TIA=\angle HAI+\angle ICB$

Vậy đpcm tương đương với:

$\angle HID=\angle HAI+\angle DIK$

$\Leftrightarrow \angle AHI=\angle DIK$ (1)

Ta có $I,B,C,J$ nằm trên đường tròn tâm $D$, $D$ là trung điểm của $IJ$

Mặt khác, $\angle ADK= 90^0$

Suy ra tam giác $IKJ$ cân tại $K$

Suy ra $\angle KID = \angle KJD$ (2)

$\angle HAI = \angle JAK$, $\frac{AH}{AI}=\frac{AJ}{AK}$ nên tam giác $AHI$ đồng dạng với tam giác $AJK$

Suy ra $ \angle AHI=\angle AJK$ (3)

Từ (1), (2), (3), ta có đpcm

Để chứng minh $\angle BJH=\angle CJK$, ta cần chứng minh:

$\angle JHI= \angle JKI$

$\Leftrightarrow IHC=\angle DKJ\Leftrightarrow \angle AHI=\angle DIK$ @~> đpcm


Bài viết đã được chỉnh sửa nội dung bởi LNH: 23-11-2014 - 19:53


#76
Juliel

Juliel

    Thượng úy

  • Thành viên
  • 1240 Bài viết

 

Bài 34: Cho dãy hữu hạn $a_0,a_1,\cdots,a_N$ thỏa mãn $a_1=a_N=0$ và $$a_{i+1}-2a_i+a_{i-1}=a_i^2,\, \forall i=1,2,3,\cdots,N-1$$

Chứng minh $a_i\le 0,\; \forall i=0,1,2,\cdots,n$.

 

Lời giải :

Nếu $0=a_1=a_{N}=\max\left \{ a_1,a_2,...,a_{N} \right \}$ thì hiển nhiên là $a_i\leq 0,\;\forall i=\overline{1,N}$.

Nếu $a_j=\max\left \{ a_1,a_2,...,a_{N} \right \}> a_1=a_{N}=0$ thì có $a_{j} \geq a_{j+1}$ và $a_j \geq a_{j-1}$. Khi đó thì :

$$a_{j}^2=(a_{j+1}-a_{j})+(a_{j-1}-a_j)\leq 0$$

Và điều này thì mâu thuẫn. Ta có đpcm.


Bài viết đã được chỉnh sửa nội dung bởi namcpnh: 09-12-2014 - 23:51

Đừng rời xa tôi vì tôi lỡ yêu người mất rồi !
 

Welcome to My Facebook !


#77
Karl Heinrich Marx

Karl Heinrich Marx

    Sĩ quan

  • Thành viên
  • 321 Bài viết

 

Vậy $g$ cũng đơn ánh, ta cũng có $g$ đơn điệu, từ $2)$ suy ra :

$$f(g(x))=x\,\,\,\Rightarrow \,\,\, 2g(x)-g(g(x))=x \\ \Rightarrow g(g(x))-g(x)=g(x)-x,\,\,\,\, (*)$$

Kí hiệu $g^{(n)}(x)=g(g(....(x)...))$ ( $n$ dấu $($ và $)$ ), lúc đó dễ thấy $g^{(n)}(x)-g^{(n-1)}(x)=g^{(n-1)}(x)-g^{(n-2)}(x)=.....=g(x)-x$, nên $$g^{(n)}(x)-x=n.(g(x)-x)\,\,\,\Rightarrow \,\,\, g^{(n)}(x)=n.(g(x)-x)+x$$

$\bullet$ TH1 : Nếu $g$ đơn điệu giảm, lúc đó với $x<y$ thì $g^{(2k+1)}(x)>g^{(2k+1)}(y),\, g^{(2k)}(x)<g^{(2k)}(y) \,\,\forall k\in \mathbb{Z}^{+}$

$\Rightarrow \,\, (2k+1).(g(x)-x)+x > (2k+1).(g(y)-y)+y \,\, , \,\,  2k.(g(x)-x)+x < 2k.(g(y)-y)+y\,\,\forall k\in \mathbb{Z}^{+}$

Thành thử ra $g(x)-x=g(y)-y$ (Vì nếu $g(x)-x<g(y)-y$ thì bđt thứ nhất sai với $k$ đủ lớn, ngược lại thì bđt 2 sai với $k$ đủ lớn).

Vậy $g(x)-x=g(y)-y\forall x,y\in \mathbb{R}$ mà tồn tại $x_0$ để $g(x_0)=x_0$ nên $g(x)=x\forall x\in \mathbb{R}$ từ đó ta có $f$.

$\bullet$ TH2 : Nếu đơn điệu tăng, làm tương tự như trên thì với $x<y$ ta luôn có $k.(g(x)-x)+x <k.(g(y)-y)+y\,\,\forall k\in \mathbb{Z}^{+}$, từ đó suy ra $g(x)-x\leq g(y)-y\forall x<y$.

 

Thay vào 2) $x$ bởi $f(x)$ và sử dụng $f(x)$ đơn ánh thì ta có $2f(x)-f(f(x))=x$ chứng minh giống bạn Whjte Shadow ở trên thì suy ra $f(x)-x \le f(y)-y$ với $x<y$. Từ đây ta có thể thấy nếu tồn tại $a,b$ mà $f(a)=a,f(b)=b$ thì $f(x)=x$ với mọi $x \in (a,b)$. Giả sử tồn tại $x_1$ mà $f(x_1) \ne x_1$ với $x_1<x_0$ khi đó sẽ phải tồn tại một số $x_2$ nhỏ nhất sao cho $f(x_2)=x_2$ khi đó thì $f(x)<x$ với mọi $x<x_2$.

Từ $1)$ và $2)$ có thể suy ra $f(x)$ là song ánh, xét một số $y_0<x_2$ và đặt $a_n=f^{-n}(y_0)$ thì dãy này tăng nghiêm ngặt và bị chặn trên bởi $x_2$ nên sẽ hội tụ tại $l$ mà $lim a_n=lim f^{n}(l)=l+lim (n(f(l)-l)) \Rightarrow lim f(l)-l=0 \Rightarrow l=x_2$ khi $n \rightarrow \infini $

$f$ liên tục và vì $f^n(x_2)=x_2>y_0$ nên điều này mâu thuẫn nên $f(x)=x$ với mọi $x \le x_0$

Chứng minh tương tự với $x \ge x_0$ ta suy ra $f(x)=x$ với mọi $x$

P/s Whjte Shadow: bài tổ hợp 16 đường thẳng 66 điểm ở trước em nên chỉ ra tính khả thi của việc dựng được 27 điểm có đúng 3 đường thẳng đi qua. Nếu tìm được số điểm max có thể dựng thỏa mãn thì chắc có thể mở rộng lên tổng quát.



#78
pndpnd

pndpnd

    Trung sĩ

  • Thành viên
  • 164 Bài viết

Bài 42: Cho đa thức $f(x)=ax^3+bx^2+cx+d$ với $a,b,c,d$ thuộc $R$ thỏa mãn $f(x)\leqslant 1$ với mọi x thuộc R và $|x|\leqslant 1$. Chứng minh với   mọi x thuộc R và $|x|\leqslant 1$ ta có $|dx^3+cx^2+bx+a|\leqslant 4$.

** Hình học:

Bài 43: Cho $(O;R)$ và $(O';R')$ với R khác R' cắt nhau tại 2 điểm phân biệt $A$ và $B$. Đường thẳng $d$ tiếp xúc với $(O)$ tại $P$, với $(O')$ tại $Q$. Gọi $M$ và $N$ là hình chiếu của $P$ và $Q$ tương ứng trên $OO'$. $AM$ giao với (O) tại $M'$ , $AN$ giao với $(O')$ tại $N'$ ( M' và N' khác A). Chứng minh $M',N',B$ thẳng hàng.


Bài viết đã được chỉnh sửa nội dung bởi pndpnd: 24-11-2014 - 21:12


#79
WhjteShadow

WhjteShadow

    Thượng úy

  • Phó Quản lý Toán Ứng dụ
  • 1323 Bài viết

Bài 37: Cho các số nguyên $x,y,z$ với $x>2,y>1,z>0$ thỏa mãn:

$x^y+1=z^2$

Gọi $p$ là số ước nguyên tố phân biệt của $x$, $q$ là số ước nguyên tố phân biệt của $y$. Chứng minh rằng: $p \geq q+2$

===============================

P/s: dạo này topic thiếu hình quá nhỉ :(

Ý tưởng là ta sẽ chỉ ra với mỗi ước nguyên tố của $y$ ta sẽ có 1 ước nguyên tố của $x$ hay gần giống như vậy, rồi chỉ thêm 2 ước là đc.

Ta xét 2 trường hợp
$\star$ Nếu $y$ chẵn thì $x^y$ và $z^2$ là 2 số chính phương và là 2 số tự nhiên liên tiếp $\Rightarrow$ mâu thuẫn do $x>2,y>1,z>0$

$\star$ Nếu $y$ lẻ, lúc đó $x^y=(z-1)(z+1)$. Lại xét 2 trường hợp tiếp :

$\bullet$ nếu $z$ chẵn, vậy $(z+1;z-1)=1$ nên $z+1=a^y,z-1=b^y$ với $a,b$ cùng lẻ $\Rightarrow a^y-b^y=2$ (mâu thuẫn vì $a-b\geq 2, y>1$ )

$\bullet$ nếu $z$ lẻ, lúc đó $(z+1;z-1)=2$, $x$ chẵn nên ta đặt viết $x=t.2^{k}$ với $t$ lẻ. Lại xét tiếp 2 trường hợp T_T

${\color{Pink} \star} \left\{\begin{matrix} z+1=2a^y\\ z-1=2^{ky-1}b^y \end{matrix}\right.$ với $(a;b)=1$, $a,b$ cùng lẻ, $a.b=x$

$\Rightarrow a^y-2^{ky-2}.b^y=1\Rightarrow 2^{ky-2}.b^y=a^y-1$.

Giả sử khi phân tích tiêu chuẩn, $y$ có $q$ ước nguyên tố là $p_1,p_2,...,p_q$. Lúc đó $\forall i=\overline{1;q}$ thì :

$a^{p_i}-1 | a^y-1 =2^{ky-2}.b^y\Rightarrow \frac{a^{p_i}-1}{a-1} | b^y$ (Có điều này vì dễ thấy $\frac{a^{p_i}-1}{a-1}\equiv p_i\pmod{2}$ là 1 số lẻ). 

Mặt khác, với thuật chia $Euclide$, ta có $(a^m-1;a^n-1)=a^{(m;n)}-1$, vậy nên với $i\neq j, \, i,j\in [1;q]$ thì $(a^{p_i}-1;a^{p_j}-1)=a-1$ hay $\left(\frac{a^{p_i}-1}{a-1};\frac{a^{p_j}-1}{a-1}\right)=1$.

Từ các điều trên ta có $b^y$ có $q$ ước dương $>1$ và đôi một nguyên tố cùng nhau, thành thử ra $b^y$ có $q$ ước nguyên tố phân biệt, hay $b$ cũng có $q$ ước nguyên tố phân biệt và cùng lẻ. Mà $(a;b)=1$ và $x$ có thêm ước nguyên tố là 2 mà $y$ không có nên nếu $a\neq 1$ thì $x$ sẽ có $\geq q+2$ ước nguyên tố ngay. Ta chỉ cần xét trường hợp $b=1$, nhưng lúc đó $a^y-1=2^{ky-2}$ hay $a-1|2^{ky-2}\Rightarrow \, a-1$ cũng là 1 lũy thừa $\geq 1$ của 2 $\Rightarrow \, a-1=2^r\Rightarrow (2^r+1)^y-1=2^{ky-2}\Rightarrow (2^r+1)^{y-1}+(2^r+1)^{y-2}+...+1=2^{ky-2-r}$ mà $(2^r+1)^{y-1}+(2^r+1)^{y-2}+...+1$ lẻ do $y$ lẻ nên $ky-2-r=0$ hay $a^y-1=a-1$ (mâu thuẫn vì $y>1,a>1$).

${\color{Pink} \star} \left\{\begin{matrix} z+1=2^{ky-1}a^y\\ z-1=2b^y \end{matrix}\right.$ với $(a;b)=1$, $a,b$ cùng lẻ, $a.b=x$, làm hoàn toàn tương tự với để ý $(b^{p_i}+1;b^{p_j}+1)=b+1$ ($p_i,p_j$ cùng lẻ) ta cũng có điều phải chứng minh.

==================

@anh Cường : Không hiểu sao em không thích tổng quát hóa bài toán lắm @@ (chắc do lười hì hì :P).

=====================

Bài 44 : Cho tam giác $ABC$ nội tiếp $(O)$, $E,F$ lần lượt thuộc cạnh $AC,AB$. $M,N,P$ lần lượt là trung điểm $EF,BE,CF$. $Q$ là hình chiếu của $O$ lên $EF$. Chứng minh rằng $M,N,P,Q$ đồng viên.


Bài viết đã được chỉnh sửa nội dung bởi WhjteShadow: 24-11-2014 - 21:20

“There is no way home, home is the way.” - Thich Nhat Hanh

#80
hoangtubatu955

hoangtubatu955

    Sĩ quan

  • Thành viên
  • 429 Bài viết

Bài 42: Cho đa thức $f(x)=ax^3+bx^2+cx+d$ với $a,b,c,d$ thuộc $R$ thỏa mãn $f(x)\leqslant 1$ với mọi x thuộc R và $|x|\leqslant 1$. Chứng minh với   mọi x thuộc R và $|x|\leqslant 1$ ta có $|dx^3+cx^2+bx+a|\leqslant 4$.

** Hình học:

Bài 43: Cho $(O;R)$ và $(O';R')$ với R khác R' cắt nhau tại 2 điểm phân biệt $A$ và $B$. Đường thẳng $d$ tiếp xúc với $(O)$ tại $P$, với $(O')$ tại $Q$. Gọi $M$ và $N$ là hình chiếu của $P$ và $Q$ tương ứng trên $OO'$. $AM$ giao với (O) tại $M'$ , $AN$ giao với $(O')$ tại $N'$ ( M' và N' khác A). Chứng minh $M',N',B$ thẳng hàng.

BÀi này khá quen, từ giả thiết thì ta chứng minh được:
$ f(x)=4x^3-3x$ luôn, như vậy ta có điều phải chứng minh!


Bài viết đã được chỉnh sửa nội dung bởi hoangtubatu955: 24-11-2014 - 21:51






Được gắn nhãn với một hoặc nhiều trong số những từ khóa sau: vmo2015

1 người đang xem chủ đề

0 thành viên, 1 khách, 0 thành viên ẩn danh